Limits of Integration for Joint Distribution problems

Click For Summary
The discussion focuses on finding the marginal densities of X and Y for a joint distribution defined over a specific region. The joint density function is determined to be fxy(x,y) = 4/3, with integration limits established based on the region defined by 0 ≤ y ≤ 1 - x² and -1 ≤ x ≤ 1. The limits of integration for the marginal densities are explained, with y ranging from 0 to 1 - x² for fixed x, and x ranging from -√(1 - y) to √(1 - y) for fixed y. The integration confirms the uniform distribution, resulting in a total probability of 1. This method illustrates the relationship between the joint distribution and the marginal distributions effectively.
trap101
Messages
339
Reaction score
0
Suppose that (X,Y) is uniformly distributed over the regiondefined by 0≤ y ≤ 1-x2
and -1≤ x ≤ 1.

a) find the marginal densities of X and Y


Attempted solution:

So first I have to find the joint density function which ends up being fxy(x,y) = 3/4

and then from that I would solve for the marginal densities. Since there was a solution I was able to do these things, but my issue is finding the limits of integration.

in finding the joint density, why did they use -1 to 1 as the limit of integration and (1-x2) to find the joint density function. Then to find the marginal densities, they used 0 to 1-x2 to find the marginal density of X and ± (1-y)1/2 to find the marginal density of Y. How and why did these limits occur?
 
Physics news on Phys.org
I assume you took a "multi-variable Calculus" course before this course. That is what is being used here. You are given that the region is "defined by 0≤ y ≤ 1-x2
and -1≤ x ≤ 1." To cover that region, x has to go from -1 to 1 and, for each x, y goes from 0 to 1- x^2[/tex]. Since you are also given that this is a <b>uniform</b> probability distribution, a constant over the region, whicy we can call &quot;M&quot;, and so the integral over the entire region must be <br /> \int_{x=-1}^1\int_{y= 0}^{1- x^2} M dydx= 1<br /> Which gives M= 4/3. That&#039;s what you did isn&#039;t it?<br /> <br /> Now, look at the &quot;inner integral&quot;:<br /> \frac{4}{3}\int_{y= 0}^{1- x^2} dy<br /> That is the integral with respect to y for fixed x. It is the probability of y for fixed x which is what is <b>meant</b> by &quot;marginal probability&quot;.<br /> <br /> Now, look at the region described by &quot;-1\le x\le 1&quot; and &quot;0\le y\le 1- x^2. That is the region under the parabola y= 1- x^2 between x= -1 and x= 1. Clearly y can take on values as low as 0 or as high as 1. <b>For each y</b>, then x can go from the parabola on the left to the parabola on the right. Since the parabola is given by y= 1- x^2, x^2= 1- y and x= \pm\sqrt{1- y}. The left side is, of course, x= -\sqrt{1- y} and the right side is x= +\sqrt{1- y}. <br /> <br /> That is, if you were to integrate<br /> \int_{y= 0}^1\int_{x=-\sqrt{1- y}}^{\sqrt{1- y}}\frac{4}{3}dxdy<br /> You would get &quot;1&quot; again. Removing the outside integral gives the &quot;marginal probabilty&quot; of x for a given y:<br /> \int_{x= -\sqrt{1- y}}^{\sqrt{1- y}}\frac{4}{3}dx
 
Question: A clock's minute hand has length 4 and its hour hand has length 3. What is the distance between the tips at the moment when it is increasing most rapidly?(Putnam Exam Question) Answer: Making assumption that both the hands moves at constant angular velocities, the answer is ## \sqrt{7} .## But don't you think this assumption is somewhat doubtful and wrong?

Similar threads

Replies
9
Views
2K
Replies
6
Views
2K
  • · Replies 3 ·
Replies
3
Views
1K
  • · Replies 13 ·
Replies
13
Views
2K
  • · Replies 5 ·
Replies
5
Views
2K
  • · Replies 15 ·
Replies
15
Views
2K
  • · Replies 3 ·
Replies
3
Views
3K
  • · Replies 4 ·
Replies
4
Views
1K
  • · Replies 3 ·
Replies
3
Views
3K
Replies
7
Views
1K